Multivariate Normal : expectation of X given Y is doubly-truncated












2














Let $(X, Y)$ be distributed as a multivariate normal with parameters



$$
mu =
begin{bmatrix}
mu_X \ mu_Y
end{bmatrix}
qquad
Sigma =
begin{bmatrix}
sigma_X^2 & sigma_{XY} \ sigma_{XY} & sigma_Y^2
end{bmatrix}.
$$



I would like to calculate $E(X | y_1 < Y < y_2)$, where $y_1$ and $y_2$ are constants.



From Wikipedia, I have managed to work out that
$$
E(X | Y > y_1) = mu_X + frac{sigma_{XY}}{sigma_Y} left[frac{phileft(frac{y_1 - mu_y}{sigma_Y}right)}{1 - Phileft(frac{y_1 - mu_y}{sigma_Y}right)}right]\
E(X | Y < y_2) = mu_X - frac{sigma_{XY}}{sigma_Y} left[frac{phileft(frac{y_2 - mu_y}{sigma_Y}right)}{Phileft(frac{y_2 - mu_y}{sigma_Y}right)}right]
,$$

where $phi(cdot)$ and $Phi(cdot)$ are, respectively, the p.d.f. and the c.d.f. of the Normal distribution. I could not figure out how to calculate $E(X | y_1 < Y < y_2)$, though.



I have searched for an answer in similar posts from the Stack Exchange network, but I couldn't get a clue from them that would solve this issue. For the record, here are some of them:




  1. Conditional expectation in the multivariate normal distribution

  2. Expectation of conditional normal distribution

  3. Conditional expectation of bivariate normal

  4. https://math.stackexchange.com/q/2807096/83294










share|cite|improve this question


















  • 1




    See stats.stackexchange.com/questions/356023/….
    – StubbornAtom
    4 hours ago










  • @StubbornAtom, thank you. I am studying the answers there and will post one here (since my question is different and the solution to it is but a step used in the answers posted there) ASAP.
    – Waldir Leoncio
    3 hours ago
















2














Let $(X, Y)$ be distributed as a multivariate normal with parameters



$$
mu =
begin{bmatrix}
mu_X \ mu_Y
end{bmatrix}
qquad
Sigma =
begin{bmatrix}
sigma_X^2 & sigma_{XY} \ sigma_{XY} & sigma_Y^2
end{bmatrix}.
$$



I would like to calculate $E(X | y_1 < Y < y_2)$, where $y_1$ and $y_2$ are constants.



From Wikipedia, I have managed to work out that
$$
E(X | Y > y_1) = mu_X + frac{sigma_{XY}}{sigma_Y} left[frac{phileft(frac{y_1 - mu_y}{sigma_Y}right)}{1 - Phileft(frac{y_1 - mu_y}{sigma_Y}right)}right]\
E(X | Y < y_2) = mu_X - frac{sigma_{XY}}{sigma_Y} left[frac{phileft(frac{y_2 - mu_y}{sigma_Y}right)}{Phileft(frac{y_2 - mu_y}{sigma_Y}right)}right]
,$$

where $phi(cdot)$ and $Phi(cdot)$ are, respectively, the p.d.f. and the c.d.f. of the Normal distribution. I could not figure out how to calculate $E(X | y_1 < Y < y_2)$, though.



I have searched for an answer in similar posts from the Stack Exchange network, but I couldn't get a clue from them that would solve this issue. For the record, here are some of them:




  1. Conditional expectation in the multivariate normal distribution

  2. Expectation of conditional normal distribution

  3. Conditional expectation of bivariate normal

  4. https://math.stackexchange.com/q/2807096/83294










share|cite|improve this question


















  • 1




    See stats.stackexchange.com/questions/356023/….
    – StubbornAtom
    4 hours ago










  • @StubbornAtom, thank you. I am studying the answers there and will post one here (since my question is different and the solution to it is but a step used in the answers posted there) ASAP.
    – Waldir Leoncio
    3 hours ago














2












2








2







Let $(X, Y)$ be distributed as a multivariate normal with parameters



$$
mu =
begin{bmatrix}
mu_X \ mu_Y
end{bmatrix}
qquad
Sigma =
begin{bmatrix}
sigma_X^2 & sigma_{XY} \ sigma_{XY} & sigma_Y^2
end{bmatrix}.
$$



I would like to calculate $E(X | y_1 < Y < y_2)$, where $y_1$ and $y_2$ are constants.



From Wikipedia, I have managed to work out that
$$
E(X | Y > y_1) = mu_X + frac{sigma_{XY}}{sigma_Y} left[frac{phileft(frac{y_1 - mu_y}{sigma_Y}right)}{1 - Phileft(frac{y_1 - mu_y}{sigma_Y}right)}right]\
E(X | Y < y_2) = mu_X - frac{sigma_{XY}}{sigma_Y} left[frac{phileft(frac{y_2 - mu_y}{sigma_Y}right)}{Phileft(frac{y_2 - mu_y}{sigma_Y}right)}right]
,$$

where $phi(cdot)$ and $Phi(cdot)$ are, respectively, the p.d.f. and the c.d.f. of the Normal distribution. I could not figure out how to calculate $E(X | y_1 < Y < y_2)$, though.



I have searched for an answer in similar posts from the Stack Exchange network, but I couldn't get a clue from them that would solve this issue. For the record, here are some of them:




  1. Conditional expectation in the multivariate normal distribution

  2. Expectation of conditional normal distribution

  3. Conditional expectation of bivariate normal

  4. https://math.stackexchange.com/q/2807096/83294










share|cite|improve this question













Let $(X, Y)$ be distributed as a multivariate normal with parameters



$$
mu =
begin{bmatrix}
mu_X \ mu_Y
end{bmatrix}
qquad
Sigma =
begin{bmatrix}
sigma_X^2 & sigma_{XY} \ sigma_{XY} & sigma_Y^2
end{bmatrix}.
$$



I would like to calculate $E(X | y_1 < Y < y_2)$, where $y_1$ and $y_2$ are constants.



From Wikipedia, I have managed to work out that
$$
E(X | Y > y_1) = mu_X + frac{sigma_{XY}}{sigma_Y} left[frac{phileft(frac{y_1 - mu_y}{sigma_Y}right)}{1 - Phileft(frac{y_1 - mu_y}{sigma_Y}right)}right]\
E(X | Y < y_2) = mu_X - frac{sigma_{XY}}{sigma_Y} left[frac{phileft(frac{y_2 - mu_y}{sigma_Y}right)}{Phileft(frac{y_2 - mu_y}{sigma_Y}right)}right]
,$$

where $phi(cdot)$ and $Phi(cdot)$ are, respectively, the p.d.f. and the c.d.f. of the Normal distribution. I could not figure out how to calculate $E(X | y_1 < Y < y_2)$, though.



I have searched for an answer in similar posts from the Stack Exchange network, but I couldn't get a clue from them that would solve this issue. For the record, here are some of them:




  1. Conditional expectation in the multivariate normal distribution

  2. Expectation of conditional normal distribution

  3. Conditional expectation of bivariate normal

  4. https://math.stackexchange.com/q/2807096/83294







conditional-expectation multivariate-normal






share|cite|improve this question













share|cite|improve this question











share|cite|improve this question




share|cite|improve this question










asked 4 hours ago









Waldir Leoncio

1,39252234




1,39252234








  • 1




    See stats.stackexchange.com/questions/356023/….
    – StubbornAtom
    4 hours ago










  • @StubbornAtom, thank you. I am studying the answers there and will post one here (since my question is different and the solution to it is but a step used in the answers posted there) ASAP.
    – Waldir Leoncio
    3 hours ago














  • 1




    See stats.stackexchange.com/questions/356023/….
    – StubbornAtom
    4 hours ago










  • @StubbornAtom, thank you. I am studying the answers there and will post one here (since my question is different and the solution to it is but a step used in the answers posted there) ASAP.
    – Waldir Leoncio
    3 hours ago








1




1




See stats.stackexchange.com/questions/356023/….
– StubbornAtom
4 hours ago




See stats.stackexchange.com/questions/356023/….
– StubbornAtom
4 hours ago












@StubbornAtom, thank you. I am studying the answers there and will post one here (since my question is different and the solution to it is but a step used in the answers posted there) ASAP.
– Waldir Leoncio
3 hours ago




@StubbornAtom, thank you. I am studying the answers there and will post one here (since my question is different and the solution to it is but a step used in the answers posted there) ASAP.
– Waldir Leoncio
3 hours ago










1 Answer
1






active

oldest

votes


















3














you should use the same approach in point 3. the conditional expectation
$$E(X|Y)=mu_X+sigma_{XY}frac{Y-mu_Y}{sigma_Y^2}$$
Then take the expectation of the RHS of this expression given $y_1<Y<y_2$. The only random variable is $Y$ and this has conditional expectation of
$$E(Y|y_1<Y<y_2)=mu_Y+sigma_Yleft[frac{phileft(frac{y_1-mu_Y}{sigma_Y}right)-phileft(frac{y_2-mu_Y}{sigma_Y}right)}{Phileft(frac{y_2-mu_Y}{sigma_Y}right)-Phileft(frac{y_1-mu_Y}{sigma_Y}right)}right]
$$

Plugging this in gives you
$$E(X|y_1<Y<y_2)=mu_X+frac{sigma_{XY}}{sigma_Y}left[frac{phileft(frac{y_1-mu_Y}{sigma_Y}right)-phileft(frac{y_2-mu_Y}{sigma_Y}right)}{Phileft(frac{y_2-mu_Y}{sigma_Y}right)-Phileft(frac{y_1-mu_Y}{sigma_Y}right)}right]
$$



this also contains your two answers as special cases $y_1to-infty$ and $y_2toinfty$






share|cite|improve this answer





















  • Thank you for your insightful answer, it helped me with a subsequential issue I was having over here.
    – Waldir Leoncio
    3 hours ago











Your Answer





StackExchange.ifUsing("editor", function () {
return StackExchange.using("mathjaxEditing", function () {
StackExchange.MarkdownEditor.creationCallbacks.add(function (editor, postfix) {
StackExchange.mathjaxEditing.prepareWmdForMathJax(editor, postfix, [["$", "$"], ["\\(","\\)"]]);
});
});
}, "mathjax-editing");

StackExchange.ready(function() {
var channelOptions = {
tags: "".split(" "),
id: "65"
};
initTagRenderer("".split(" "), "".split(" "), channelOptions);

StackExchange.using("externalEditor", function() {
// Have to fire editor after snippets, if snippets enabled
if (StackExchange.settings.snippets.snippetsEnabled) {
StackExchange.using("snippets", function() {
createEditor();
});
}
else {
createEditor();
}
});

function createEditor() {
StackExchange.prepareEditor({
heartbeatType: 'answer',
autoActivateHeartbeat: false,
convertImagesToLinks: false,
noModals: true,
showLowRepImageUploadWarning: true,
reputationToPostImages: null,
bindNavPrevention: true,
postfix: "",
imageUploader: {
brandingHtml: "Powered by u003ca class="icon-imgur-white" href="https://imgur.com/"u003eu003c/au003e",
contentPolicyHtml: "User contributions licensed under u003ca href="https://creativecommons.org/licenses/by-sa/3.0/"u003ecc by-sa 3.0 with attribution requiredu003c/au003e u003ca href="https://stackoverflow.com/legal/content-policy"u003e(content policy)u003c/au003e",
allowUrls: true
},
onDemand: true,
discardSelector: ".discard-answer"
,immediatelyShowMarkdownHelp:true
});


}
});














draft saved

draft discarded


















StackExchange.ready(
function () {
StackExchange.openid.initPostLogin('.new-post-login', 'https%3a%2f%2fstats.stackexchange.com%2fquestions%2f385423%2fmultivariate-normal-expectation-of-x-given-y-is-doubly-truncated%23new-answer', 'question_page');
}
);

Post as a guest















Required, but never shown

























1 Answer
1






active

oldest

votes








1 Answer
1






active

oldest

votes









active

oldest

votes






active

oldest

votes









3














you should use the same approach in point 3. the conditional expectation
$$E(X|Y)=mu_X+sigma_{XY}frac{Y-mu_Y}{sigma_Y^2}$$
Then take the expectation of the RHS of this expression given $y_1<Y<y_2$. The only random variable is $Y$ and this has conditional expectation of
$$E(Y|y_1<Y<y_2)=mu_Y+sigma_Yleft[frac{phileft(frac{y_1-mu_Y}{sigma_Y}right)-phileft(frac{y_2-mu_Y}{sigma_Y}right)}{Phileft(frac{y_2-mu_Y}{sigma_Y}right)-Phileft(frac{y_1-mu_Y}{sigma_Y}right)}right]
$$

Plugging this in gives you
$$E(X|y_1<Y<y_2)=mu_X+frac{sigma_{XY}}{sigma_Y}left[frac{phileft(frac{y_1-mu_Y}{sigma_Y}right)-phileft(frac{y_2-mu_Y}{sigma_Y}right)}{Phileft(frac{y_2-mu_Y}{sigma_Y}right)-Phileft(frac{y_1-mu_Y}{sigma_Y}right)}right]
$$



this also contains your two answers as special cases $y_1to-infty$ and $y_2toinfty$






share|cite|improve this answer





















  • Thank you for your insightful answer, it helped me with a subsequential issue I was having over here.
    – Waldir Leoncio
    3 hours ago
















3














you should use the same approach in point 3. the conditional expectation
$$E(X|Y)=mu_X+sigma_{XY}frac{Y-mu_Y}{sigma_Y^2}$$
Then take the expectation of the RHS of this expression given $y_1<Y<y_2$. The only random variable is $Y$ and this has conditional expectation of
$$E(Y|y_1<Y<y_2)=mu_Y+sigma_Yleft[frac{phileft(frac{y_1-mu_Y}{sigma_Y}right)-phileft(frac{y_2-mu_Y}{sigma_Y}right)}{Phileft(frac{y_2-mu_Y}{sigma_Y}right)-Phileft(frac{y_1-mu_Y}{sigma_Y}right)}right]
$$

Plugging this in gives you
$$E(X|y_1<Y<y_2)=mu_X+frac{sigma_{XY}}{sigma_Y}left[frac{phileft(frac{y_1-mu_Y}{sigma_Y}right)-phileft(frac{y_2-mu_Y}{sigma_Y}right)}{Phileft(frac{y_2-mu_Y}{sigma_Y}right)-Phileft(frac{y_1-mu_Y}{sigma_Y}right)}right]
$$



this also contains your two answers as special cases $y_1to-infty$ and $y_2toinfty$






share|cite|improve this answer





















  • Thank you for your insightful answer, it helped me with a subsequential issue I was having over here.
    – Waldir Leoncio
    3 hours ago














3












3








3






you should use the same approach in point 3. the conditional expectation
$$E(X|Y)=mu_X+sigma_{XY}frac{Y-mu_Y}{sigma_Y^2}$$
Then take the expectation of the RHS of this expression given $y_1<Y<y_2$. The only random variable is $Y$ and this has conditional expectation of
$$E(Y|y_1<Y<y_2)=mu_Y+sigma_Yleft[frac{phileft(frac{y_1-mu_Y}{sigma_Y}right)-phileft(frac{y_2-mu_Y}{sigma_Y}right)}{Phileft(frac{y_2-mu_Y}{sigma_Y}right)-Phileft(frac{y_1-mu_Y}{sigma_Y}right)}right]
$$

Plugging this in gives you
$$E(X|y_1<Y<y_2)=mu_X+frac{sigma_{XY}}{sigma_Y}left[frac{phileft(frac{y_1-mu_Y}{sigma_Y}right)-phileft(frac{y_2-mu_Y}{sigma_Y}right)}{Phileft(frac{y_2-mu_Y}{sigma_Y}right)-Phileft(frac{y_1-mu_Y}{sigma_Y}right)}right]
$$



this also contains your two answers as special cases $y_1to-infty$ and $y_2toinfty$






share|cite|improve this answer












you should use the same approach in point 3. the conditional expectation
$$E(X|Y)=mu_X+sigma_{XY}frac{Y-mu_Y}{sigma_Y^2}$$
Then take the expectation of the RHS of this expression given $y_1<Y<y_2$. The only random variable is $Y$ and this has conditional expectation of
$$E(Y|y_1<Y<y_2)=mu_Y+sigma_Yleft[frac{phileft(frac{y_1-mu_Y}{sigma_Y}right)-phileft(frac{y_2-mu_Y}{sigma_Y}right)}{Phileft(frac{y_2-mu_Y}{sigma_Y}right)-Phileft(frac{y_1-mu_Y}{sigma_Y}right)}right]
$$

Plugging this in gives you
$$E(X|y_1<Y<y_2)=mu_X+frac{sigma_{XY}}{sigma_Y}left[frac{phileft(frac{y_1-mu_Y}{sigma_Y}right)-phileft(frac{y_2-mu_Y}{sigma_Y}right)}{Phileft(frac{y_2-mu_Y}{sigma_Y}right)-Phileft(frac{y_1-mu_Y}{sigma_Y}right)}right]
$$



this also contains your two answers as special cases $y_1to-infty$ and $y_2toinfty$







share|cite|improve this answer












share|cite|improve this answer



share|cite|improve this answer










answered 3 hours ago









probabilityislogic

18.9k36384




18.9k36384












  • Thank you for your insightful answer, it helped me with a subsequential issue I was having over here.
    – Waldir Leoncio
    3 hours ago


















  • Thank you for your insightful answer, it helped me with a subsequential issue I was having over here.
    – Waldir Leoncio
    3 hours ago
















Thank you for your insightful answer, it helped me with a subsequential issue I was having over here.
– Waldir Leoncio
3 hours ago




Thank you for your insightful answer, it helped me with a subsequential issue I was having over here.
– Waldir Leoncio
3 hours ago


















draft saved

draft discarded




















































Thanks for contributing an answer to Cross Validated!


  • Please be sure to answer the question. Provide details and share your research!

But avoid



  • Asking for help, clarification, or responding to other answers.

  • Making statements based on opinion; back them up with references or personal experience.


Use MathJax to format equations. MathJax reference.


To learn more, see our tips on writing great answers.





Some of your past answers have not been well-received, and you're in danger of being blocked from answering.


Please pay close attention to the following guidance:


  • Please be sure to answer the question. Provide details and share your research!

But avoid



  • Asking for help, clarification, or responding to other answers.

  • Making statements based on opinion; back them up with references or personal experience.


To learn more, see our tips on writing great answers.




draft saved


draft discarded














StackExchange.ready(
function () {
StackExchange.openid.initPostLogin('.new-post-login', 'https%3a%2f%2fstats.stackexchange.com%2fquestions%2f385423%2fmultivariate-normal-expectation-of-x-given-y-is-doubly-truncated%23new-answer', 'question_page');
}
);

Post as a guest















Required, but never shown





















































Required, but never shown














Required, but never shown












Required, but never shown







Required, but never shown

































Required, but never shown














Required, but never shown












Required, but never shown







Required, but never shown







Popular posts from this blog

Understanding the information contained in the Deep Space Network XML data?

Ross-on-Wye

Eastern Orthodox Church